WILL GIVE BRAINLEST


Robert is ordering jerseys for the baseball team

A local company charges $5,25 for each jersey and $45 to create the design

An online company charges $7.75 for each jersey and $20 to create the design

For which number of jerseys is the cost the same at both companies?

A)26 jerseys
B)1 jersey
С)5 jerseys
D)10 jerseys

Answers

Answer 1

Answer: D) 10 jerseys

Step-by-step explanation:

Let j represent the number of jerseys and C for total cost

Local company eqn: C = 5.25j + 45

Online company eqn: C = 7.75j + 20

Set both sides equal to solve:

5.25j + 45 = 7.75j + 20

5.25j - 7.75j = 20 -45

- 2.5j = -25

[tex]\frac{-2.5j}{-2.5}=\frac{-25}{-2.5}[/tex]

j = 10

Therefore, the cost is the same for both companies when 10 jerseys are sold


Related Questions

Choose the angle which is represented by the name.

R

Answers

Answer:

Option B

Step-by-step explanation:

Angles can be named by only the middle point's name. Option B would be correct in this case, because the point in the center of the angle is point R.

If the original quantity is 10 and the new quantity is 3​, what is the percent​ increase?

Answers

Answer:

30% Increase

Step-by-step explanation:

We Know 3 is 30% of 10 and 10+3=13 so we know that 13 is a 30% increase of 10.

Hopes This Helps :)

Solve by elimination show each step will rate you 5 stars

Answers

Answer:Multiply X + Y by -6 so you can cancel 6x by -6x. Then you get 3y+24. Then divide by 3 and get Y=8 Then solve for X. Use one of the equations and solve for X. I would do X+8=16. Then minus 8 and get 8. So X=8

Step-by-step explanation:

With that being said the coordinate pair would be (8,8)

Which expression is equivalent to (15aºb²c34) (3a166-29cº) for all values of a, b, and
c where the expression is defined?

Answers

The answer is 45a16c34b27

Which relation is a function?

Answers

Answer:

only the v-shaped graph is a function

Step-by-step explanation:

the circle, the vertical line, and the sideways parabola would not pass the 'vertical line test' which is where, if a vertical line is drawn through the graph then the line would not intersect the graph in more than one point

the set $s = \$ contains the first $50$ positive integers. after the multiples of 2 and the multiples of 3 are removed, how many integers remain in the set $s$?

Answers

Clearly, |S| = 50.

Count the multiples of 2 between 1 and 50:

⌊50/2⌋ = ⌊25⌋ = 25

(where ⌊x⌋ denotes the "floor of x", or the largest integer that is smaller than or equal to x; in other words, round down to the nearest integer)

Count the multiples of 3 between 1 and 50:

⌊50/3⌋ ≈ ⌊16.667⌋ = 16

Since LCM(2, 3) = 6, the sets of multiples of 2 and multiples of 3 have some overlap. Count the multiples of 6 between 1 and 50:

⌊50/6⌋ ≈ ⌊8.333⌋ = 8

Then by the inclusion/exclusion principle, we remove from S

25 + 16 - 8 = 33

elements, so that the new set S contains 50 - 33 = 17 elements.

What is the distance between -9and -2 on the number line

Answers

Answer:

7 units

Step-by-step explanation:

Calculate the distance using the absolute value function

| - 9 - (- 2) | = | - 9 + 2 | = | - 7 | = 7

or

| - 2 - (- 9) | = | - 2 + 9 | = | 7 | = 7

The distance between -9 and -2 on the number line is -7.

How i can usea the distribute property on this equation?

Answers

Answer:

9(3-2r)

Step-by-step explanation:

27-18r the greatest common factor between 27 and 18 is 9. therefore, we can factor 9 out.

9(3-2r)

Hope this helps.

Answer:

9(3-2r)

Step-by-step explanation:

Need some help on this question please

Answers

[tex]2(4x+1)<3(2x-3)\\\\\implies 8x +2 < 6x -9\\\\\implies 8x <6x -9 -2\\\\\implies 8x -6x < -11\\\\\implies 2x<-11\\\\\implies x< -\dfrac{11}2\\\\\\\text{Interval,} ~ \left(-\infty, -\dfrac{11}2 \right)[/tex]

Answer:

x<3.5

Step-by-step explanation:

multiply 2 with 4x and 1 to get 8x + 2 < 3(2x-3). Then multiply 3 by 2x and -3 to get 8x + 2 < 6x- 9. Then subtract -6x on both sides to get 2x + 2< -9. Next subtract 2 from -9 so x is by itself and then you get 2x < 7. Finally divide 2 on each side so you leave x byitself completely. You get x > 3.5

I hope u get it right!

A fast food place was selling 5 boxes of chicken nuggets for 25.50 a competing restaurant was selling 1 box of chicken strips for 5.50 which is better priced per box, 5 boxes chicken nuggets for 25.50 or 1 box of chicken strips for 5.50

Answers

Answer:

The chicken which is sold for 5 boxes for 25.50 is better priced.

Step-by-step explanation:

If you buy five boxes of the chicken from the place where it is 5.50 a box, buying five will be 27.50, which costs more than the place with the 25.50 price for five boxes.

A potter makes cups and saucers in a factory. He is paid $1.44 per
batch of cups and $1.20
per
batch of saucers. What is his gross pay if
he makes 9 batches of cups and 11 batches of saucers in one day?

Answers

Answer:

4

Step-by-step explanation:

what is the length of XY​

Answers

Answer:

15

Step-by-step explanation:

Help me, please! I'll give 20 points.

Answers

Answer:

A = 97.5

Step-by-step explanation:

Since A & B are supplementary, the general equation would be:

180 - 82.5 = A

A = 97.5

What is 1+1 equal? Explain in paragraph form! 5 or more sentences!

Answers

1 + 1 = 2. If you have 1 of something and you get another one then you have two.

I hope this helps give you a start

Given each function and domain, find the range values. y=3/5x+2; domain: {-10,0,5}

Answers

Given the function

[tex]y=\dfrac{3}{5}x+2[/tex]

and its domain [tex]\{-10,0,5\}[/tex], its range is [tex]\{-4,2,5\}[/tex]

The domain of a function is the set of all possible values that we can input to the function.

The range of a function is the set of all possible values that a function can output.

For the function

[tex]y=\dfrac{3}{5}x+2[/tex]

if the domain is the set [tex]\{-10,0,5\}[/tex], we can get the range by substituting each value in the domain into the function. The resulting set of values gives the range of the function.

The range will then be

[tex]\left\{\dfrac{3}{5}(-10)+2, \dfrac{3}{5}(0)+2,\dfrac{3}{5}(5)+2\right\}\\\\=\left\{3(-2)+2, 3(0)+2, 3(1)+2\right\}\\\\=\left\{-4, 2, 5\right\}\\\\[/tex]

Learn more about function domain and range here https://brainly.com/question/13824428

Jonita determined that the distance between her apartment and the nearest hospital was more than 1,600 meters but less than 1,900 meters. Which of the following could be a reasonable estimate for
the distance between her house and the hospital?

O 1.7 x 102 meters
O 1.8 x 103 meters
O 1.7 x 10-3 meters
O 1.8 x 10-2 meters

Answers

Answer: 1.8 x 103 meters

Step-by-step explanation:

The average of the two extremes is 1,700 meters.  This would be 1.7x10^3 in scientific notation.  None of the four options include this number.  But three stand out as total losers.  We can convert the answer options to see the difference:

O 1.7 x 102 meters  =    170 meters      [Loser]

O 1.8 x 103 meters  =  1800 meters

O 1.7 x 10-3 meters =  0.0017 meters   [Loser]

O 1.8 x 10-2 meters =  0.018 meters     [Loser]

Which statement is true when looking at a black ball?

Answers

Answer:

Only the primary color wavelengths are being absorbed by the ball.

Answer:Which statement is true when looking at a black ball?

Group of answer choices

Only the primary color wavelengths are being reflected by the ball.

All color wavelengths are being reflected by the ball.

All color wavelengths are being absorbed by the ball.

Only the primary color wavelengths are being absorbed by the ball.

Step-by-step explanation:

Help me please ;-;

For the function y = 4x – 2, what is the value of the function when x is equal to -3?

A. -10
B. 14
C. -14
D. 10

Answers

Answer:

-14

Step-by-step explanation:

Answer:

C

Step-by-step explanation:

y=4x - 2                                  x= -3

y=4(-3) - 2

y= -12 - 2

y= -14

What is the volume of a pencil

Answers

around 19.05 centimeters or 7.4 inches

ANS WER AS SOON AS POSSIBLE
AND I WILL MARK U AS BRAINLIST

Answers

False

Take RHS = 6 x 105 + 6 x 102 = 6 x 100000 + 6 x 100 = 600000 + 600 = 600600 ≠ LHS Hence, RHS ≠ LHS

Hoped that helped!

Answer:

False

Step-by-step explanation:

use PEMDAS.

since there is no perenthesis you start with exponents.

10^5=100000

10^2=100

6*100,000+6*100

Next, you go to multipliation

600,000+600

Finally, you move on to addition

600,600

1. You are setting up a shelf display filled with present
boxes and other decor. Each shelf has a length of 116
inches and a width of 30 inches. If there are 5 shelves,
what is the total area you can use for the decor?
2. Buddy is figuring out how long his snowflake chain
around the whole store needs to be. Use the diagram
below to find the perimeter of the store! Type your
work and solution in the box under the diagram.

Answers

Answer:

1. 17,400 in²

2. ???

Step-by-step explanation:

1. If the the length of each shelf is 116 inches and the width is 30 inches then 1 shelf has 3480 in² of space. Since that's for 1 shelf and we have 5 shelves you multiply 3480 by 5 to get 17400.

2. You didn't give any diagram so I can't really help here but just add up all the side of the shape it gave you. You'll probably have to do some math to find missing sides.

Using Trig to Find a Side

Answers

Sin42= opp/hyp

Sin42=6.4/x

X= 6.4/sin42 = 9.564

X= 9.56

I hope that helps and sorry if my answer turned out to be wrong

The value of the length x in a given triangle by using the trigonometry formula would be 9.6 units.

Given that,

A triangle UVW is shown in the image with sides and angles,

vw = 6.4

uv = x

And, ∠wuv = 42°

Used the trigonometry formula which states that,

[tex]\text{ sin x} = \dfrac{\text{ Opposite}}{\text{ Hypotenuse}}[/tex]

Here, we have;

An angle ∠wuv = 42°.

So, we get;

[tex]\text{ sin 42} = \dfrac{\text{ vw}}{\text{ uv}}[/tex]

Substitute given values,

[tex]\text{ sin 42} = \dfrac{\text{ 6.4}}{\text{ x}}[/tex]

Since, sin (42°) = 0.67

Hence,

[tex]0.67 = \dfrac{6.4}{x}[/tex]

Multiply by x on both sides,

[tex]0.67 \times x = 6.4[/tex]

Divide both sides by 0.67, we get;

[tex]x = \dfrac{6.4}{0.67}[/tex]

[tex]x = 9.55[/tex]

Round to the nearest tenth,

[tex]x = 9.6[/tex]

So, the value of x is 9.6 units.

To learn more about the triangle visit;

brainly.com/question/1058720

#SPJ6

Need help doing this

Answers

Answer:

huh I'm confused I dont know

PLS HELP ME I'M SO CONFUSED!!!!!!

Answers

Answer:

a = -3, b = 2.5, c = - 1/2

Step-by-step explanation:

1)

[tex]3^a=1/27[/tex][tex]3^a=1/3^3[/tex][tex]3^a=3^{-3}[/tex][tex]a=-3[/tex]

2)

[tex]3^b=9\sqrt{3}[/tex][tex]3^b=3^2*3^{1/2}[/tex][tex]3^b=3^{2+1/2}[/tex][tex]3^b=3^{2.5}[/tex][tex]b=2.5[/tex]

3)

[tex]3^c=1/\sqrt{3}[/tex][tex]3^c=1/3^{1/2}[/tex][tex]3^c=3^{-1/2}[/tex][tex]c = - 1/2[/tex]

which expression is equivalent 2(t-4)+1?

Answers

Answer:

(Answer below)

Step-by-step explanation:

2t - 8 + 1

this is the expanded form of 2(t-4)+1 therefore, it is the same as 2(t-4)+1

how I got this: 2 x t = 2t           2 x 4 = 8           2t + 8 +1

10. Mrs. Taylor wants to lay new carpet in the entrance and hallway of her home as shown below. The carpet she selected sells for $2.50 a square foot. How much will it cost to lay the carpet in the entrance and hallway?

Answers

Don’t u just multiply $2.50 to 2…

Plz Heeeeelp! 25 POINTS! I'm so desperate I made two of these worth 25 points!

Answers

Answer:

The answer is B, f(x)=5(2/5)^(z)

Step-by-step explanation:

I did the graphs for all of them and graph B was identical to the one in the picture.

Answer:

  (b)  f(x) = 5(2/5)^x

Step-by-step explanation:

All of the offered choices agree that the multiplier of the exponential factor is 5. When x=1, the exponential factor must have a value of 2/5 in order to make the function value be 2 at that point.

That is, ...

  f(1) = 5(2/5)^1 = 5(2/5) = 2

You can get there either by using x=1 in each of the offered answer choices, or by observing that the ratio between f(1) and f(0) is 2/5.

  f(x) = 5(2/5)^x

does (0,0) make the equation y = x true​

Answers

True because its true maybe but just answer true

. What is the fraction form of $0.25?

Answers

Answer:

1/4

Step-by-step explanation:

Since 0.25 is 1/4 of 1



Suppose that 600 yards of fencing material are available to fence in

two adjacent rectangular corrals sharing a common fence. Find the

dimensions of the corrals that will enclose a maximum area (Please help)

Answers

Answer:

Length = 150 yards

Width = 100 yards

Step-by-step explanation:

We want 600 yards of fencing that will result in the largest 2 fenced corrals, sharing a common border.

It will take the shape of a rectangle, with a dividing fence down the center.

Let W and L,  Width and Length of the larger enclosure.

See attachment.

W= Area of the larger enclosure.

The perimeter is 2W + 2L.

The dividing fence is 1W

We know that we only have 600 yards of fence, so:

2W + 2L + 1W = 600 yards

Area = W x L

---

3W + 2L  = 600    (yards)

2L  = 600 -3W

L = (600-3W)/2

L = 300 -(3/2)W

---

Use this expression in the Area calculation:

Area = W x L

Area = W x (300 -(3/2)W)

Area = 300W -(3/2)W^2)

To find the maximum area, take the first derivative and set to zero to find the value of W that results in the greatest area.

Area' = 300 -2(3/2)W)

0 = 300 - 3W

3W = 300

W = 100 yards

Since 3W + 2L  = 600

L = (600 - 3W)/2

L = (600 - 3(100))/2

L = 150 yards

Area = 150*100 = 15,000 yards^2

 

Other Questions
Multiply and write the result in Scientific notation (3 x 10^5) (2 x 10^2)? After your first meeting, if the lawyer you are considering says "i will win this case. In fact, ive never lost a case like this. " then you should feel _____. A. Excited b. Ambivalent c. Wary d. Confident. Solve the proportion:11/10 = 22/x If a firm hires 10 workers at $9 per hour each and the 11th worker will be hired only if the wage rate falls to $8 per hour, the marginal wage rate must be _____.Multiple Choice$2$2$2.20$2.20 why does marty stuart wear a black rag around his neck Which function is shown in the graph? Marhannah must make a costume for the school play. She needs a piece of fabric thats is 8/3 yards long and 3/2 yards wide. What is the area of the piece of fabric Marhannah needs? What is the product of 44(4-7)(4)?-16O-1 -12-1008DONE i got a important question for yallfor the last week now ive been having sudden pains in my stomach, it hurts really bady and im trying to avoid going to the doctors cause i don't trust them at all. I just need to know if anyone knows any other way i can get rid of these sudden sharp pains in my stomach what identifies a cell or a range of cells on a worksheet, and tells excel where to look for values or data you to use in a formula Its About drive its about powerwe stay hungry we devour Which number is closest to 1/2 3 + 4y= 412 + 16y = 8 how do you evaluate numbers in math? ___________ (you ever act) in a school play before this one? a text setting with many notes set to a syllable is called Bearing in mind that those two tissue samples removed from Henrietta were not removed in an attempt totreat her cancer, but rather purely for purposes of research, was it wrong for the doctor to remove thesample tissue in the first place? Was it wrong for Dr. Gey to collect those samples for the purpose oftrying to grow them in controlled conditions? Does the end - i.e., the immeasurable benefit to humankindresulting from those tissue samples - justify the means - i.e., removing tissue from a person without theirconsent or knowledge? Factor. 9(p-1)+(p-1)^2 How does appropriation help artists express themselves? Ten sentence of Italian language